به محفل ریاضی ایرانیان خوش آمدید! لطفا برای استفاده از تمامی امکانات عضو شوید
سایت پرسش و پاسخ ریاضی
+1 امتیاز
66 بازدید
در دبیرستان و دانشگاه توسط mansour (532 امتیاز)

فرض کنید $$a_{1}, a_{2}, a_{3} ,..., a_{n} $$ اعداد حقیقی مثبت باشند نابرابری زیر را ثابت کنید:
$$4 ( \sum _ {i < j} \frac{1}{ a_{i} + a_{j} } )^{2} \leq \binom{n}{2} \sum _ {i < j} \frac{1}{ a_{i} a_{j} } $$

1 پاسخ

0 امتیاز
توسط قاسم شبرنگ (3,000 امتیاز)
انتخاب شده توسط mansour
 
بهترین پاسخ

نامساوی کوشی_شوارتز :

$( \sum _{i=1}^nx_iy_i)^2 \leq (\sum _{i=1}^nx_i^2)(\sum _{i=1}^ny_i^2)$

نامساوی میانگین هندسی_حسابی:

$ \frac{a_i+a_j}{2} \geq \sqrt{a_ia_j} \equiv ( \frac{2}{a_i+a_j} )^2 \leq \frac{1}{a_ia_j} $

حالا این نامساوی های زیبا را برای دنبالۀ $x=( \frac{1}{a_i+a_j} )_{i< j}$ و دنبالۀ ثابت $y=(1)$ که هر کدام $ \binom{n}{2} $ جمله دارند (چرا؟) به کار بگیرید:

$ 4(\sum_{i< j} \frac{1}{a_i+a_j} )^2 \leq 4(\sum _{i=1}^{ \binom{n}{2} }1^2)(\sum _{i< j}(\frac{1}{a_i+a_j}) ^2)= \binom{n}{2}(\sum _{i< j}(\frac{2}{a_i+a_j}) ^2)$

$= \binom{n}{k} \sum _{i< j} \frac{1}{a_ia_j} $

$ \Box $

تساوی چه وقت اتفاق می افتد؟

توسط mansour (532 امتیاز)
وقتی که تمام اعداد حقیقی مثبت داده شده برابر باشند.

حمایت مالی

کانال تلگرام محفل ریاضی
امروز : تاریخ شمسی اینجا نمایش داده می‌شود
...